Maximum Likelihood Decoding of Linear Codes on Binary-Input Memoryless Channels

Given the linear code C\mathcal{C} ={(0,0,0,0,0,0),= \{(0,0,0,0,0,0), (0,1,0,1,0,1),(0,1,0,1,0,1), (1,0,0,0,1,0),(1,0,0,0,1,0), (1,1,0,1,1,1)}(1,1,0,1,1,1)\} and a codeword x=\textbf{x} = (0,1,0,1,0,1)(0,1,0,1,0,1) transmitted over the BEC(ϵ)BEC(\epsilon). Choose the correct option for probability of error if received vector is y\textbf{y} =(0,1,?,1,0,?)= (0,1,?,1,0,?), where ?? represents erasures.
Explanation

Explanation

Explanation

Explanation

Explanation

Explanation

Explanation

Explanation

Consider a quaternary erasure channel QEC(ϵ)QEC(\epsilon) which takes four input symbols, {0,1,2,3}\{0, 1, 2, 3\}, where the channel erases the transmitted symbol with probability ϵ\epsilon. Choose the valid pair of transmitted and received vectors for transmission over QEC(ϵ)QEC(\epsilon) channel.
Explanation

Explanation

Explanation

Explanation

Explanation

Explanation

Explanation

Explanation

Consider the code C\mathcal{C} ={(0,0,0,0,0,0),= \{(0,0,0,0,0,0), (1,1,0,0,1,1),(1,1,0,0,1,1), (1,1,1,0,1,1),(1,1,1,0,1,1), (0,0,1,0,0,0)}(0,0,1,0,0,0)\}. Let the vector x\textbf{x} =(1,1,1,0,1,1)= (1,1,1,0,1,1) be transmitted over BSCBSC. Suppose we know that  exactly one bit \textit{ exactly one bit } is flipped. Which of the following options can not be a potential received vector?
Explanation

Explanation

Explanation

Explanation

Explanation

Explanation

Explanation

Explanation

Suppose we are given a code C\mathcal{C} ={(0,0,0),= \{(0,0,0), (1,1,1)}(1,1,1)\}. One codeword from this code, C\mathcal{C}, is transmitted over Binary Symmetric Channel BSC(p)BSC(p) with bit-flip probability pp. Calculate the probability of error for incorrect decoding using Maximum Likelihood Decoder.
Explanation

Explanation

Explanation

Explanation

Explanation

Explanation

Explanation

Explanation

Consider the code C\mathcal{C} ={(0,0,0),(1,0,1),=\{(0,0,0),(1,0,1), (0,1,1),(1,1,0)}(0,1,1),(1,1,0)\}. Let x\textbf{x} be a codeword (in bipolar form) is being transmitted over the AWGN channel, with mean 00 and variance 11. Suppose the receiver receives y\textbf{y} =(103,0.5,1)= (-10^3,0.5,1). Choose the correct Maximum Likelihood estimate for the transmitted codeword.
Explanation

Explanation

Explanation

Explanation

Explanation

Explanation

Explanation

Explanation